very indirect ans

This topic has expert replies
Junior | Next Rank: 30 Posts
Posts: 12
Joined: Tue Apr 10, 2007 6:41 am
Location: bangalore

very indirect ans

by hedonistatbreak » Wed Jun 13, 2007 9:33 pm
A researcher discovered that people who have low levels of immune-system activity tend
to score much lower on tests of mental health than do people with normal or high
immune-system activity. The researcher concluded from this experiment that the immune
system protects against mental illness as well as against physical disease.
The researcher’s conclusion depends on which of the following assumptions?
A. High immune-system activity protects against mental illness better than normal
immune-system activity does.
B. Mental illness is similar to physical disease in its effects on body systems.
C. People with high immune-system activity cannot develop mental illness.
D. Mental illness does not cause people’s immune-system activity to decrease.
E. Psychological treatment of mental illness is not as effective as is medical treatment.

What do you think will be the answer to the above question?

Master | Next Rank: 500 Posts
Posts: 105
Joined: Mon Sep 18, 2006 12:34 pm
Location: OH
Thanked: 7 times
GMAT Score:780

by mschling52 » Thu Jun 14, 2007 4:41 am
I think it's D. The researcher is saying that since mental illness and low immune system activity tend to occur together, it must be that mental illness is the result of the low immune system.

However, it could easily be that the causation is in the opposite direction and the low immune system activity is caused by the mental illness. But, for his conclusion to be valid, the researcher must assume that this is not the case.

Senior | Next Rank: 100 Posts
Posts: 91
Joined: Tue May 01, 2007 4:04 pm

by Nisha1218 » Thu Jun 14, 2007 5:43 pm
I would think its A, but not 100% sure.

User avatar
Community Manager
Posts: 789
Joined: Sun Jan 28, 2007 3:51 pm
Location: Silicon valley, California
Thanked: 30 times
Followed by:1 members

by jayhawk2001 » Thu Jun 14, 2007 8:25 pm
Another vote for D.

B is a close one as it seems to bridge the gap (or comparison) between
mental illness and physical disease. However, the conclusion is better
boosted by D.

Junior | Next Rank: 30 Posts
Posts: 12
Joined: Tue Apr 10, 2007 6:41 am
Location: bangalore

by hedonistatbreak » Thu Jun 14, 2007 10:18 pm
Yeah D is the right answer.

FYI below is the explanation:

The researcher concludes from the association of low immune-system activity with low mental-health sores that, in effect, immune system activity can inhibit mental illness. If, 80
contrary to D, mental illness can depress immune-system activity, the association mentioned does not support the researcher’s conclusion. So D must be assumed. Normal immune-system activity could protect against mental illness without high-immune system activity offering increased protection or prevention, contrary to what A and C state, so neither
of A and C is assumed. The conclusion does not depend on there being a similarity between mental and physical illness or a difference in treatments, so B and E are not assumed.


I feel the answer is very indirect as the examinee has to think other way round in the way what are the effects of mental illness on immune systems instead of immune systems ability to prevent mental illness. What a crappy question!!!
i wish such quesions do not appear while i try my fate :lol:

User avatar
Junior | Next Rank: 30 Posts
Posts: 24
Joined: Sat May 12, 2007 11:51 pm
Thanked: 1 times

by rocky » Thu Jun 14, 2007 10:41 pm
POE works better in this question it leads to only D
Getting into it

Junior | Next Rank: 30 Posts
Posts: 12
Joined: Tue Apr 10, 2007 6:41 am
Location: bangalore

by hedonistatbreak » Thu Jun 14, 2007 11:54 pm
hey rocky .. wat is POE ?

Senior | Next Rank: 100 Posts
Posts: 91
Joined: Tue May 01, 2007 4:04 pm

by Nisha1218 » Fri Jun 15, 2007 6:12 am
Process
Of
Elimination

Master | Next Rank: 500 Posts
Posts: 120
Joined: Sun Apr 13, 2014 3:55 am

by john83.amar » Tue May 03, 2016 2:52 am
I like explanation for D